0 votes
in Sets, relations and functions by (90.1k points)
edited by
\(\lim _{n \rightarrow \infty} \frac{3}{n}\left\{1+\sqrt{\frac{n}{n+3}}+\sqrt{\frac{n}{n+6}}+\sqrt{\frac{n}{n+9}}+\ldots+\sqrt{\frac{n}{n+3(n-1)}}\right\}\)
(A) does not exit
(B) is 1
(C) is 2
(D) is 3

3 Answers

0 votes
by (90.1k points)
Ans : (C)
Hint \(: \operatorname{Lt}_{n \rightarrow \infty} \frac{3}{n} \sum_{r=0}^{n-1} \sqrt{\frac{1}{1+3\left(\frac{r}{n}\right)}}\)
$$
=3 \int_{0}^{1} \frac{d x}{\sqrt{1+3 x}}=2[\sqrt{1+3 x}]_{0}^{1}=2
$$
0 votes
by
cialis 5mg drug <a href="https://ordergnonline.com/">buy tadalafil 5mg pills</a> online ed meds
0 votes
by
cialis 5mg ca <a href="https://ordergnonline.com/">order cialis 40mg for sale</a> online ed pills
...